Fórum de Matemática
DÚVIDAS? Nós respondemos!

Um Fórum em Português dedicado à Matemática
Data/Hora: 28 mar 2024, 16:48

Os Horários são TMG [ DST ]




Fazer Nova Pergunta Responder a este Tópico  [ 5 mensagens ] 
Autor Mensagem
MensagemEnviado: 30 jun 2017, 15:06 
Offline

Registado: 25 jan 2014, 19:24
Mensagens: 158
Localização: Joinville - Sc
Agradeceu: 100 vezes
Foi agradecido: 1 vez(es)
Poderia por gentileza detalhar como a equação \(t^2-\frac{3}{2}t-1\)


pode ser passada para a forma \((t-\frac{3}{4})^2-\frac{25}{16}\)??


Topo
 Perfil  
 
MensagemEnviado: 01 jul 2017, 01:08 
Offline

Registado: 08 jan 2015, 18:39
Mensagens: 930
Localização: Campo Grande - MS - Brasil
Agradeceu: 14 vezes
Foi agradecido: 475 vezes
Boa noite!

Foi utilizada uma técnica de completar quadrados:
É simples:
\(t^2-at\)

O que queremos é obter um termo que deixe aparecer um quadrado perfeito. Então, comparemos:
\((t-k)^2=t^2-2kt+k^2\)

Veja que na primeira equação temos 'at' e na segunda temos '2kt'. Então, podemos imaginar que o k é a metade de a, certo?

Façamos o seguinte:
\(t^2-at=t^2-2(a/2)t=t^2-2(a/2)t+(a/2)^2-(a/2)^2\)

Veja que gerei o termo que faltava para completar... somei e subtraí ao mesmo tempo, e forma a não modificar a equação. Agora:
\(t^2-2(a/2)t+(a/2)^2-(a/2)^2=(t-(a/2))^2-(a/2)^2\)

Vamos tentar executar para o que nos mostrou:
\(t^2-\frac{3}{2}t-1\\t^2-2\left(\frac{3}{4}\right)t-1\\t^2-2\left(\frac{3}{4}\right)t+\frac{3}{4}^2-\frac{3}{4}^2-1\\\left(t-\frac{3}{4}\right)^2-\frac{9}{16}-1\\\left(t-\frac{3}{4}\right)^2-\frac{25}{16}\)

Espero ter ajudado!

_________________
Baltuilhe
"Nós somos o que fazemos repetidamente. Excelência, então, não é um modo de agir, é um hábito." Aristóteles


Topo
 Perfil  
 
MensagemEnviado: 01 jul 2017, 17:27 
Offline

Registado: 25 jan 2014, 19:24
Mensagens: 158
Localização: Joinville - Sc
Agradeceu: 100 vezes
Foi agradecido: 1 vez(es)
Boa tarde!

Primeiramente muito obrigado!

Só ficou uma dúvida!

Como sumiu o termo \(2\frac{3}{4}t\)?


Topo
 Perfil  
 
MensagemEnviado: 01 jul 2017, 21:30 
Offline

Registado: 08 jan 2015, 18:39
Mensagens: 930
Localização: Campo Grande - MS - Brasil
Agradeceu: 14 vezes
Foi agradecido: 475 vezes
Boa tarde!

Editei a resposta para corrigir um erro.
O termo é o seguinte:
\((x+a)^2=x^2+2ax+b^2\)

Veja que ao desenvolver aparece um termo 2ax. Quando colocado dentro dos parênteses, não há mais o '2'. Entendeu?

Espero ter ajudado!

_________________
Baltuilhe
"Nós somos o que fazemos repetidamente. Excelência, então, não é um modo de agir, é um hábito." Aristóteles


Topo
 Perfil  
 
MensagemEnviado: 01 jul 2017, 22:30 
Offline

Registado: 25 jan 2014, 19:24
Mensagens: 158
Localização: Joinville - Sc
Agradeceu: 100 vezes
Foi agradecido: 1 vez(es)
Entendi o processo, porém, ainda acho que estou com dificuldade.
O caso abaixo pode ilustrar o problema que estou tendo.
Como seria no caso abaixo?
\(\int \frac{dx}{3sinx-4cosx}= \int \frac{\frac{2dt}{1+t^2}}{3(\frac{2t}{1+t^2})-4(\frac{1-t^2}{1+t^2})}=\int \frac{\frac{2dt}{1+t^2}}{\frac{6t-4+t^2}{1+t^2}}=\int \frac{2dt}{2(2t^2+3t-2)}=\frac{1}{2}\int \frac{dt}{t^2+\frac{3}{2}t-2}=\)

daí para a frente eu não estou conseguindo acertar.

a resposta do livro é:
\(\frac{1}{5}ln{{\frac{2tan\frac{x}{2}-1}{tan\frac{x}{2}+2}}}+C\)

obrigado pela ajuda!


Topo
 Perfil  
 
Mostrar mensagens anteriores:  Ordenar por  
Fazer Nova Pergunta Responder a este Tópico  [ 5 mensagens ] 

Os Horários são TMG [ DST ]


Quem está ligado:

Utilizadores a ver este Fórum: Nenhum utilizador registado e 57 visitantes


Criar perguntas: Proibído
Responder a perguntas: Proibído
Editar Mensagens: Proibído
Apagar Mensagens: Proibído
Enviar anexos: Proibído

Pesquisar por:
Ir para: